Challenging Inequalities

Algebra Level 4

Positive real numbers a a , b b , and c c are such that a b c = 2 abc=2 . If the minimum value of

1 a 3 ( b + c ) + 1 b 3 ( c + a ) + 1 c 3 ( a + b ) \large \frac{1}{a^3(b+c)} + \frac{1}{b^3(c+a)} + \frac{1}{c^3(a+b)} is equal to p q 3 \sqrt[3]{\frac{p}{q}} , where p p and q q are coprime positive integers. Enter 2 p 2p as the answer.


The answer is 54.

This section requires Javascript.
You are seeing this because something didn't load right. We suggest you, (a) try refreshing the page, (b) enabling javascript if it is disabled on your browser and, finally, (c) loading the non-javascript version of this page . We're sorry about the hassle.

2 solutions

Chew-Seong Cheong
Jan 10, 2018

Consider c y c 1 a 3 ( b + c ) = c y c 1 a 2 a b + c a = c y c 1 a 2 a b c c + a b c b = 1 2 c y c 1 a 2 1 b + 1 c \displaystyle \sum_{cyc} \frac 1{a^3(b+c)} = \sum_{cyc} \frac {\frac 1{a^2}}{ab+ca} = \sum_{cyc} \frac {\frac 1{a^2}}{\frac {abc}c+\frac {abc}b} = \frac 12 \sum_{cyc} \frac {\frac 1{a^2}}{\frac 1b+\frac 1c} . Using Titu's lemma and then AM-GM inequality :

1 2 ( 1 a 2 1 b + 1 c + 1 b 2 1 c + 1 a + 1 c 2 1 a + 1 b ) 1 2 ( ( 1 a + 1 b + 1 c ) 2 2 ( 1 a + 1 b + 1 c ) ) = 1 4 ( 1 a + 1 b + 1 c ) 3 4 1 a b c 3 = 27 128 3 \begin{aligned} \frac 12 \left( \frac {\frac 1{a^2}}{\frac 1b+\frac 1c} + \frac {\frac 1{b^2}}{\frac 1c+\frac 1a} + \frac {\frac 1{c^2}}{\frac 1a+\frac 1b} \right) & \ge \frac 12 \left(\frac {\left(\frac 1a+\frac 1b+\frac 1c\right)^2}{2\left(\frac 1a+\frac 1b+\frac 1c\right)} \right) = \frac 14 \left(\frac 1a+\frac 1b+\frac 1c\right) \ge \frac 34 \sqrt[3]{\frac 1{abc}} = \sqrt[3]{\frac {27}{128}} \end{aligned}

Equality occurs when a = b = c = 2 3 a=b=c=\sqrt[3]2 . Therefore, 2 p = 2 × 27 = 54 2p = 2\times 27 = \boxed{54} .

A perfect solution. By the way, the question is a modified form of the Problem 2 taken from IMO 1995

Shreyansh Mukhopadhyay - 3 years, 5 months ago

Log in to reply

Glad that you like it.

Chew-Seong Cheong - 3 years, 5 months ago

l e t a = b = c = 2 3 . T h a n f ( a , b , c ) m i n = 27 128 3 = p q 3 . 2 p = 2 27 = 54. C h e c k e d f o r v a l u e s n e a r 2 3 a n d o t h e r v a l u e s t o s e e t h a t i t i s t h e m i n i m u m . let~a=b=c=\sqrt[3]{2}.\\ Than~f(a,b,c)_{min}=\sqrt[3]{\dfrac{27}{128}}=\sqrt[3]{\dfrac p q}.\\ 2p=2*27=54.\\ Checked ~for~ values~ near~~ \sqrt[3]2~~ and~~ other~ values~ to~ see~ that~ it~ is~ the~ minimum.

0 pending reports

×

Problem Loading...

Note Loading...

Set Loading...